§1. Bất đẳng thức

Lê Hoàng Thắng

chứng minh

2(x2y+y2z+z2y)-(xy+yz+xz)≥3

∀ x,y,z>0 và xyz=1

ai giúp mình bài trên với.

Akai Haruma
28 tháng 10 2019 lúc 19:49

Lời giải:

Do $xyz=1$ nên tồn tại $a,b,c>0$ sao cho $(x,y,z)=(\frac{a}{b}, \frac{b}{c}, \frac{c}{a})$

Khi đó bài toán trở thành:

Cho $a,b,c>0$. CMR: \(2\left(\frac{a^2}{bc}+\frac{b^2}{ca}+\frac{c^2}{ab}\right)-\left(\frac{a}{c}+\frac{b}{a}+\frac{c}{b}\right)\geq 3\)

\(\Leftrightarrow \frac{2(a^3+b^3+c^3)-(a^2b+b^2c+c^2a)}{abc}\geq 3\)

\(\Leftrightarrow 2(a^3+b^3+c^3)\geq a^2b+b^2c+c^2a+3abc(*)\)

---------------

Áp dụng BĐT AM-GM:

\(a^3+b^3+c^3\geq 3\sqrt[3]{a^3b^3c^3}=3abc(1)\)

Và:

\(\frac{a^3}{3}+\frac{a^3}{3}+\frac{b^3}{3}\geq 3\sqrt[3]{\frac{a^6b^3}{3^3}}=a^2b\)

\(\frac{b^3}{3}+\frac{b^3}{3}+\frac{c^3}{3}\geq 3\sqrt[3]{\frac{b^6c^3}{3^3}}=b^2c\)

\(\frac{c^3}{3}+\frac{a^3}{3}+\frac{a^3}{3}\geq 3\sqrt[3]{\frac{c^6a^3}{3^3}}=c^2a\)

Cộng theo vế và rút gọn \(\Rightarrow a^3+b^3+c^3\geq a^2b+b^2c+c^2a(2)\)

Lấy $(1)+(2)$ ta thu được $(*)$

Do đó ta có đpcm

Dấu "=" xảy ra khi $a=b=c$ hay $x=y=z=1$

Bình luận (0)
 Khách vãng lai đã xóa
tthnew
28 tháng 10 2019 lúc 18:53

Đặt \(\left(x;y;z\right)=\left(\frac{a'}{b'};\frac{b'}{c'};\frac{c'}{a'}\right)\).Cần chứng minh:

\(2\left(\frac{a'^2}{b'c'}+\frac{b'^2}{c'a'}+\frac{c'^2}{a'b'}\right)-\left(\frac{b'}{a'}+\frac{c'}{b'}+\frac{a'}{c'}\right)\)

Đặt \(\left(\frac{a'}{b'};\frac{b'}{c'};\frac{c'}{a'}\right)=\left(a;b;c\right)\). Bây giờ bài toán trở nên dễ dàng hơn:

Cho a, b, c > 0 thỏa mãn abc = 1. Chứng minh rằng \(2\left(ab+bc+ca\right)-\left(\frac{1}{a}+\frac{1}{b}+\frac{1}{c}\right)\ge3\)

\(\Leftrightarrow\frac{1}{a}+\frac{1}{b}+\frac{1}{c}\ge3\). Rất hiển nhiên điều này đúng theo AM-GM: \(\frac{1}{a}+\frac{1}{b}+\frac{1}{c}\ge3\sqrt[3]{\frac{1}{abc}}=3\)

Ta có điều phải chứng minh.

Is that true? Nếu nó đúng, em nghĩ bài này mấu chốt là nhìn ra cách đặt đầu tiên, và một chút may mắn:)

Bình luận (0)
 Khách vãng lai đã xóa
tthnew
30 tháng 10 2019 lúc 10:16

Đầu tiên:\(x^2y+x^2y+y^2z\ge3\sqrt[3]{x^4y^4z}=3xy\)

Tương tự các bđt còn lại và cộng theo vế sẽ suy ra được:

\(x^2y+y^2z+z^2x\ge xy+yz+zx\)

Từ đó \(VT\ge x^2y+y^2z+z^2x\ge3\sqrt[3]{\left(xyz\right)^3}=3^{\left(đpcm\right)}\)

Đơn giản nhễ:))

Bình luận (0)
 Khách vãng lai đã xóa

Các câu hỏi tương tự
ngân hồng
Xem chi tiết
Lưu Thị Thảo Ly
Xem chi tiết
Văn Quyết
Xem chi tiết
Ryan Park
Xem chi tiết
muon tim hieu
Xem chi tiết
Phan Cả Phát
Xem chi tiết
Nguyen Kim Chi
Xem chi tiết
Lục Khả Vi
Xem chi tiết
dbrby
Xem chi tiết